Is S = {(a+1,b,0)|a,b are real numbers} a Subspace of R^3?

Click For Summary
The discussion centers on whether the set S = {(a+1,b,0)|a,b are real numbers} is a subspace of R^3. Initial attempts to prove S is not a subspace include counterexamples involving scalar multiplication and vector addition that seemingly yield results outside of S. However, participants argue that these counterexamples are flawed, as the resulting vectors can still belong to S. A detailed proof is presented showing that S meets the criteria for a subspace, including closure under addition and scalar multiplication. Ultimately, the consensus is that S is indeed a subspace of R^3, contrary to the initial claim.
lkh1986
Messages
96
Reaction score
0

Homework Statement



Show that S = {(a+1,b,0)|a,b are real numbers} is NOT a subspace of R^3.



Homework Equations





The Attempt at a Solution



I take a specific counter example:

Let k = 0 inside real, and u = (1+1,1,0) inside S
ku = 0(1+1,1,0) = (0,0,0) not inside S
So, S is not a subspace.
[I can let k = 0 right? Because 0 is also a real number]

Another counter example:
Let u = v = (-1+1,1,0) inside S
u + v = (0,2,0) not inside S
So, S is not a subspace.

Which of the counter examples should I use? It seems that the first one makes more sense to me. The second one is really weird.

Thanks.
 
Physics news on Phys.org
lkh1986 said:

Homework Statement



Show that S = {(a+1,b,0)|a,b are real numbers} is NOT a subspace of R^3.



Homework Equations





The Attempt at a Solution



I take a specific counter example:

Let k = 0 inside real, and u = (1+1,1,0) inside S
ku = 0(1+1,1,0) = (0,0,0) not inside S
So, S is not a subspace.
[I can let k = 0 right? Because 0 is also a real number]

Another counter example:
Let u = v = (-1+1,1,0) inside S
u + v = (0,2,0) not inside S
So, S is not a subspace.

Which of the counter examples should I use? It seems that the first one makes more sense to me. The second one is really weird.

Thanks.

I don't think you should use either one. (0,0,0) is the case a=(-1) b=0. (0,2,0) is a=(-1) b=2. They are both is S. In fact, S IS a subspace. It's the same as the set {(a,b,0)}.
 
Dick said:
I don't think you should use either one. (0,0,0) is the case a=(-1) b=0. (0,2,0) is a=(-1) b=2. They are both is S. In fact, S IS a subspace. It's the same as the set {(a,b,0)}.

Yes, that's what I think as well. S is a subspace, because since the unknown a is a real number, so why don't I replace a+1 by another unknown, let say c, where c is also a real number.

The thing is, the answer provided says S is NOT a subspace. So it has me confused.

So, here's my steps to show S is a subspace.

(i) When a = -1, b = 0, (0,0,0) inside S.
(ii) Let vector u = (u1+1,u2,0) and vector v = (v1+1,v2,0), both inside S.
So u + v = ((u1+v1+1)+1, (u2+v2),0) also inside S.
(iii) Let k inside real and u = (u1+1,u2,0)
ku = k(u1+1,u2,0) = (ku1+k,ku2,0)= ((ku1+k-1)+1,ku2,0) also inside S.

The last part, i.e. (ku1+k,ku2,0)= ((ku1+k-1)+1,ku2,0) is a bit strange but somehow it seems okay for me.
 
lkh1986 said:
Yes, that's what I think as well. S is a subspace, because since the unknown a is a real number, so why don't I replace a+1 by another unknown, let say c, where c is also a real number.

The thing is, the answer provided says S is NOT a subspace. So it has me confused.

So, here's my steps to show S is a subspace.

(i) When a = -1, b = 0, (0,0,0) inside S.
(ii) Let vector u = (u1+1,u2,0) and vector v = (v1+1,v2,0), both inside S.
So u + v = ((u1+v1+1)+1, (u2+v2),0) also inside S.
(iii) Let k inside real and u = (u1+1,u2,0)
ku = k(u1+1,u2,0) = (ku1+k,ku2,0)= ((ku1+k-1)+1,ku2,0) also inside S.

The last part, i.e. (ku1+k,ku2,0)= ((ku1+k-1)+1,ku2,0) is a bit strange but somehow it seems okay for me.

Your proof looks good to me. I suspect somebody just wasn't thinking clearly when they wrote the problem.
 
Thanks very much for the help. Greatly appreciated. :)
 
Question: A clock's minute hand has length 4 and its hour hand has length 3. What is the distance between the tips at the moment when it is increasing most rapidly?(Putnam Exam Question) Answer: Making assumption that both the hands moves at constant angular velocities, the answer is ## \sqrt{7} .## But don't you think this assumption is somewhat doubtful and wrong?

Similar threads

  • · Replies 4 ·
Replies
4
Views
2K
  • · Replies 1 ·
Replies
1
Views
2K
  • · Replies 5 ·
Replies
5
Views
7K
  • · Replies 1 ·
Replies
1
Views
1K
  • · Replies 6 ·
Replies
6
Views
2K
  • · Replies 1 ·
Replies
1
Views
2K
  • · Replies 10 ·
Replies
10
Views
2K
  • · Replies 15 ·
Replies
15
Views
2K
  • · Replies 2 ·
Replies
2
Views
2K
Replies
8
Views
2K